TransWikia.com

LaTex math mode coding

TeX - LaTeX Asked by Emily P on February 19, 2021

This is my code, in begin{align*} why am I getting an error saying I’m missing a $?

begin{document}
%%
begin{proof*}
begin{align*}

1.; & (p land q rightarrow r, u rightarrow lnot r, u land q) &&aside{Premise}
2.; & p land q rightarrow r   &&aside{Premise} 
3.; & equiv u rightarrow lnot r     &&aside{Premise} 
4.; & equiv u land q                 &&aside{Premise} 
5.; & equiv u                         &&aside{Specialization:3} 
6.; & equiv land r                   &&aside{Modus Pones:2,4} 
7.; & equiv lnot (p land q)         &&aside{Modus Tollens:1,5} 
8.; & equiv lnot p lor lnot q      &&aside{De Morgan's: ^} 
9.; & equiv q                         &&aside{Specialization:3}
10.; & equiv lnot lnot q             &&aside{Double Negation:8}
11.; & equiv lnot p                   &&aside{Elimination:7,9}
                    
end{align*}
end{proof*}        

%%
end{document}

2 Answers

Welcome to TeX.SE!

To long for a comment ...

Since you not provide MWE, the following is based on usual use of the proof environment (see Theorems). In it is considered @Mico comment (removed are blank lines in align environment):

documentclass{article}
usepackage{amsmath,amsthm}
newcommandaside[1]{text{#1}}

begin{document}
    begin{proof} % <---
begin{align*}
    1.quad & (p land q to r, u to lnot r, u land q) 
                        &&aside{Premise}
    2.quad & p land q rightarrow r   &&aside{Premise} 
    3.quad & equiv u rightarrow lnot r     &&aside{Premise} 
    4.quad & equiv u land q                 &&aside{Premise} 
    5.quad & equiv u                         &&aside{Specialization:3} 
    6.quad & equiv land r                   &&aside{Modus Pones:2,4} 
    7.quad & equiv lnot (p land q)         &&aside{Modus Tollens:1,5} 
    8.quad & equiv lnot p lor lnot q      &&aside{De Morgan's: } 
    9.quad & equiv q                         &&aside{Specialization:3}
    10.quad & equiv lnot lnot q            &&aside{Double Negation:8}
    11.quad & equiv lnot p                  &&aside{Elimination:7,9}
        qedhere
end{align*}%
    end{proof} % <---
end{document}

enter image description here

It gives no errors! Where and how you define ˙proof*`?

Answered by Zarko on February 19, 2021

For some reason, align environment cannot process empty lines. So remove the empty lines and it should compile fine

Answered by Masum on February 19, 2021

Add your own answers!

Ask a Question

Get help from others!

© 2024 TransWikia.com. All rights reserved. Sites we Love: PCI Database, UKBizDB, Menu Kuliner, Sharing RPP